login
The OEIS is supported by the many generous donors to the OEIS Foundation.

 

Logo
Hints
(Greetings from The On-Line Encyclopedia of Integer Sequences!)
A345896 Numbers k such that k, k^2-1 and k^2+1 are all fine, where a number m is fine if its prime factors are all less than m^(1/3). 1

%I #12 Jul 01 2021 15:52:44

%S 2673,10625,12168,14651,24167,28800,32085,34162,48279,50692,59892,

%T 60928,61347,61952,64960,66125,66339,70400,71995,74704,80730,83028,

%U 89424,93024,96348,100491,108675,111475,112632,114954,117649,118048,121121,122877,127224,144925,147204,149178,150072,153340

%N Numbers k such that k, k^2-1 and k^2+1 are all fine, where a number m is fine if its prime factors are all less than m^(1/3).

%C Numbers k such that A006530(k)^3 < k, A006530(k^2-1)^3 < k^2-1, and A006530(k^2+1)^3 < k^2+1.

%H Robert Israel, <a href="/A345896/b345896.txt">Table of n, a(n) for n = 1..10000</a>

%H MathOverflow, <a href="https://mathoverflow.net/questions/396310/can-all-three-numbers-n-n2-1-n21-be-fine-as-opposed-to-coarse/396318#comment1014828_396318">Can all three numbers n, n^2-1, n^2+1 be fine (as opposed to coarse)?</a>

%e a(3) = 12168 is a term because:

%e A006530(12168) = 13 with 13^3 = 2197 < 12168,

%e 12168^2-1 = 148060223, A006530(148060223) = 283 with 283^3 = 22665187 < 148060223, and

%e 12168^2+1 = 148060225, A006530(148060225) = 293 with 293^3 = 25153757 < 148060225.

%p isfine:= proc(n) andmap(t -> t^3 < n, numtheory:-factorset(n)) end proc:

%p filter:= t -> isfine(t) and isfine(t^2-1) and isfine(t^2+1):

%p select(filter, [$1..200000]);

%Y Cf. A006530.

%K nonn

%O 1,1

%A _Robert Israel_, Jun 29 2021

Lookup | Welcome | Wiki | Register | Music | Plot 2 | Demos | Index | Browse | More | WebCam
Contribute new seq. or comment | Format | Style Sheet | Transforms | Superseeker | Recents
The OEIS Community | Maintained by The OEIS Foundation Inc.

License Agreements, Terms of Use, Privacy Policy. .

Last modified August 12 19:26 EDT 2024. Contains 375113 sequences. (Running on oeis4.)